2
$\begingroup$

I found this problem on a fb group and the author asked to use Miquel's theorem to solve it:

enter image description here

let $ABCD$ be a quadrilateral with $AB=AD$, and suppose $\angle BCD=\tfrac{1}{2}\bigl(180^\circ-\angle BAD\bigr)$.

Let $P$ be any point on the interior of the side $CD$. Let $Q=AP\cap BC$ and $R=DQ\cap BP$.

Prove that $\angle BRD=\angle BCD$.

I tried to use that theorem to solve the problem, but found no way. Do you know how?

$\endgroup$

2 Answers 2

4
$\begingroup$

The claim is equivalent to $R\in\Gamma_{BCD}$.

The circle centered at $A$ through $B$ is orthogonal to $\Gamma_{BCD}$, so $A$ is the pole of $BD$ with respect to $\Gamma_{BCD}$ and we may rephrase the problem in the following way:

Application of De La Hire's theorem

Equivalent theorem. If $DBCR$ is a cyclic quadrilateral with diagonals meeting at $P$ and $Q=BC\cap RD$, then the pole of $BD$ with respect to the circumcircle lies on the $PQ$ line.

Proof. Let $S=BD\cap CR$ be the missing vertex of the complete quadrilateral. Then $SQ=\text{pol}(P)$ and by De La Hire's theorem $S=\text{pol}(QP)$ and $Q=\text{pol}(SP)$. Since $QS,PS,BD$ are concurrent, their poles are collinear. In other terms, $A\in PQ=\text{pol}(S)$ since $S\in\text{pol}(A)$.

$\endgroup$
1
  • $\begingroup$ Great! Do you think is it possible to adapt your proof using Miquel's theorem too? Thanks $\endgroup$ Commented Sep 14 at 6:22
1
$\begingroup$

Let $K$ be the intersection of the circle through $A,B,D$ with the line $AP$ (it is easy to see that such a point always exists).

Then the quadrilateral $ABKD$ is cyclic. Note that $\angle BDA = \angle DBA=\frac{1}{2}(180-\angle BAD)$, (since the triangle is isosceles), so $\angle BDA = \angle AKB = \angle DKA $.

Since $\angle PCB = \angle PKB$, the quadrilateral $BCKP$ is cyclic. enter image description here Also, because $\angle DCQ = \angle DKQ$, the quadrilateral $QCKD$ is cyclic. enter image description here

By Miquel's theorem, there exists a unique point through which the circles $PBC$, $QDC$, and $RDP$ all intersect.
The points of intersection of $QDC$ and $PBC$ are $C$ and $K$, so one of them must also lie on $RDP$.

It cannot be $C$, because in that case we would have three collinear points on the same circle, which is impossible.
Therefore it must be $K$, and hence $RDPK$ is cyclic.

Thus $$ \angle DRP = \angle DKP = \angle DCB. $$

enter image description here

$\endgroup$
3
  • $\begingroup$ Nice answer! I have few doubts: where you use that $ \angle BCD=\tfrac{1}{2}\bigl(180^\circ-\angle BAD\bigr) $ ? $\endgroup$ Commented Sep 22 at 7:51
  • $\begingroup$ @user967210 See the edit please $\endgroup$ Commented Sep 22 at 13:58
  • $\begingroup$ so you use that fact to prove that $BCKP$ is cyclic? $\endgroup$ Commented Sep 22 at 23:00

You must log in to answer this question.

Start asking to get answers

Find the answer to your question by asking.

Ask question

Explore related questions

See similar questions with these tags.